Zero-point energy from Schroedinger equation using Ritz variaton principle

Note that there is no factor of two in front of the integral, and there is an extra factor of r^2 and a factor of sin(theta) in the integrand.The integral measure in spherical coordinates is r^2\sin\theta\,dr\,d\theta\,d\phi. Note that this is NOT the same
  • #1
Juqon
31
0

Homework Statement


Calculate the zero-point energy of the hydrogen atom using the Ritz variaton principle and the approach [itex]\psi_{\alpha}[/itex].
Hint: The stationary Schroedinger equation in spherical coordinates is ...

Homework Equations


[tex] \left\{ -\frac{\hbar^{2}}{2\mu}\left[\frac{\partial^{2}}{\partial r^{2}}+\frac{2}{r}\frac{\partial}{\partial r}\right]+\frac{\hat{\vec{L}}^{2}}{2\mu r^{2}}+V(r)\right\} \psi=E\psi [/tex][tex]

\psi_{\alpha}(\vec{r})=C\exp\left\{ -\alpha r\right\} ;C>0;\alpha>0; C,\alpha=const. [/tex] [tex]

\int_{0}^{\infty}dr\cdot r\cdot\exp\left\{ -2\alpha r\right\} =\frac{1}{4}\alpha^{2} [/tex][tex]

\int_{0}^{\infty}dr\cdot r^{2}\cdot\exp\left\{ -2\alpha r\right\} =\frac{1}{4}\alpha^{3} [/tex]



The Attempt at a Solution



[tex]
\left\{ -\frac{\hbar^{2}}{2\mu}\left[\frac{\partial^{2}}{\partial r^{2}}+\frac{2}{r}\frac{\partial}{\partial r}\right]+\frac{\hat{\vec{L}}^{2}}{2\mu r^{2}}+V(r)\right\} \psi=E\psi [/tex][tex]

\Leftrightarrow \left\{ -\frac{ \hbar^{2} }{2\mu} \left [\frac{\partial^{2}} {\partial r^{2}}+ \frac{2}{r} \frac {\partial} {\partial r} \right]+ \frac {\left(\hbar^{2} l(l+1) \right)^{2}} {2\mu r^{2}}+\frac{1} {4\pi\epsilon_{0} } \frac{qQ}{r} \cdot C \cdot \exp -\alpha r \right\} \cdot C\cdot\exp\left\{ -\alpha r\right\} = E \cdot C\cdot\exp\left\{ -\alpha r\right\} [/tex][tex]

\Leftrightarrow \left\{ -\frac{\hbar^{2}}{2\mu}\left [\alpha^{2}-\frac{2}{r}\alpha\right] +\frac{\hbar^{4}l^{2} (l^{2}+2l+1)}{2\mu r^{2}} +\frac{1}{4\pi\epsilon_{0}}\frac{qQ} {r}\right\} \cdot C\cdot\exp\left\{ -\alpha r\right\} = E \cdot C\cdot\ exp\left\{ -\alpha r\right\} [/tex][tex]

\Leftrightarrow E=\left\{ -\frac{\hbar^{2}}{2\mu}\left[\alpha^{2}-\frac{2}{r}\alpha \right]+\frac{\hbar^{4}l^{2}(l^{2}+2l+1)}{2\mu r^{2}}+\frac{1}{4\pi\epsilon_{0}} \frac{qQ}{r}\right\} [/tex]

Where do I need to integrate at all?
 
Last edited:
Physics news on Phys.org
  • #2
Your Schrodinger equation seems to be missing a plus or minus sign.

All you've done is apply the Hamiltonian to the trial function. Your estimate for E is supposed to be a constant. You still have r's and l's in it as well as the parameter alpha. You need to review what you're supposed to do to solve for the estimate of the ground state energy. This example might help you see what the general procedure is:

http://quantummechanics.ucsd.edu/ph130a/130_notes/node378.html#example:hopoly
 
  • #3
Ok, I will correct the sign, sorry.
The example was very helpful, thanks. Did I use the integral measure correctly? And is the result really that complicated?


[tex]
E_{0}=\frac{<\psi|\hat{H}|\psi>}{<\psi|\psi>}= \frac{\int_{-\infty}^{\infty} dr\cdot r^{2}\cdot\psi^{*}\left\{ -\frac{\hbar^{2}}{2\mu}\left[\alpha^{2}-\frac{2}{r}\alpha\right]\psi+\frac{\hbar^{4}l^{2}(l^{2}+2l+1)}{2\mu r^{2}}\psi+\frac{1}{4\pi \epsilon_{0} } \frac{qQ}{r}\psi\right\} }{\int_{-\infty}^{\infty}dr\psi^{*}\psi}[/tex]

[tex]


=\frac{2\int_{0}^{\infty}dr\cdot r^{2}\cdot\exp\left\{ -2\alpha r\right\} \left\{ \frac{ \hbar^{2}}{2\mu}\frac{2}{r}\alpha+\frac{ \hbar^{4}l^{2}(l^{2}+2l+1)}{2\mu r^{2}}-\frac{ \hbar^{2}}{2\mu}\alpha^{2}+\frac{1}{4\pi \epsilon_{0} } \frac{qQ}{r}\right\} }{2\int_{0}^{\infty}dr\cdot r^{2}\cdot\exp\left\{ -2\alpha r\right\} }[/tex]

[tex]


=\frac{2\int_{0}^{\infty}dr\cdot\exp\left\{ -2\alpha r\right\} \left\{ \frac{\hbar^{2}}{2\mu}2r\alpha+r^{2}\frac{\hbar^{4}l^{2}(l^{2}+2l+1)}{2\mu}-r^{2}\frac{\hbar^{2}}{2\mu}\alpha^{2}+\frac{r}{4 \pi \epsilon_{0} } \frac{qQ}{1}\right\} }{2\int_{0}^{\infty}dr\cdot r^{2}\cdot\exp\left\{ -2\alpha r\right\} }[/tex]

[tex]


=\frac{2\int dr\cdot\exp\left\{ -2\alpha r\right\} \cdot r\left\{ \frac{\hbar^{2}}{2\mu}2\alpha+\frac{1}{4\pi \epsilon_{0} } \frac{qQ}{1}\right\} +2\int dr\cdot\exp\left\{ -2\alpha r\right\} \cdot r^{2}\cdot\left(\frac{\hbar^{4}l^{2}(l^{2}+2l+1)}{2\mu}-\frac{\hbar^{2}}{2\mu}\alpha^{2}\right)}{2\int_{0}^{\infty}dr\cdot r^{2}\cdot\exp\left\{ -2\alpha r\right\} }[/tex]

[tex]


=\frac{2\cdot\frac{1}{4}\alpha^{2}\left\{ \frac{\hbar^{2}}{2\mu}2\alpha+\frac{1}{4\pi \epsilon_{0} } \frac{qQ}{1}\right\} +2\frac{1}{4}\alpha^{3}\cdot\left(\frac{\hbar^{4}l^{2}(l^{2}+2l+1)}{2\mu}-\frac{\hbar^{2}}{2\mu}\alpha^{2}\right)}{2\frac{1}{4}\alpha^{3}}[/tex]

[tex]


=\frac{\frac{1}{2}\alpha^{2}\left\{ \frac{\hbar^{2}}{2\mu}2\alpha+\frac{1}{4\pi \epsilon_{0} } \frac{qQ}{1}\right\} +\frac{1}{2}\alpha^{3}\cdot\left(\frac{\hbar^{4}l^{2}(l^{2}+2l+1)}{2\mu}-\frac{\hbar^{2}}{2\mu}\alpha^{2}\right)}{\frac{1}{2}\alpha^{3}}[/tex]

[tex]


E_{0}=\frac{1}{4\pi \epsilon_{0} } \frac{qQ}{\alpha}+\frac{ \hbar^{2}}{2\mu}2+\frac{\hbar^{4}l^{2}(l^{2}+2l+1)-\hbar^{2}\alpha^{2}}{2\mu}[/tex]

[tex]


\frac{\partial E_{0}}{\partial\alpha}=- \frac{1}{4\pi \epsilon_{0} } \frac{qQ}{\alpha^{2}}+\frac{\hbar^{4}l^{2}(l^{2}+2l+1)-\hbar^{2}\alpha}{\mu}=0\Leftrightarrow\frac{\hbar^{4}l^{2}(l^{2}+2l+1)-\hbar^{2}\alpha}{\mu}=\frac{1}{4\pi \epsilon_{0} } \frac{qQ}{\alpha^{2}}[/tex]

[tex]


\Leftrightarrow\alpha^{3}= \frac{qQ}{4\pi \epsilon_{0} } \frac{\mu}{\hbar^{4}l^{2}(l^{2}+2l+1)- \hbar^{2}}\Leftrightarrow\alpha= \sqrt[3]{\frac{qQ}{4\pi \epsilon_{0} } \frac{\mu}{\hbar^{2}\left(l^{2}(l^{2}+2l+1)-1\right)}}[/tex]

[tex]


E_{0}=\frac{1}{4\pi \epsilon_{0} } \frac{qQ}{\sqrt[3]{\frac{qQ}{4\pi \epsilon_{0} } \frac{\mu}{\hbar^{2}\left(l^{2}(l^{2}+2l+1)-1\right)}}}+ \frac{\hbar^{2}}{2\mu}2+ \frac{\hbar^{4}l^{2}(l^{2}+2l+1) - \hbar^{2}\left(\frac{qQ}{4\pi \epsilon_{0} } \frac{\mu}{\hbar^{2}\left(l^{2}(l^{2}+2l+1)-1\right)}\right)^{\frac{2}{3}}}{2\mu} [/tex]
 
  • #4
It's a bit simpler than what you have. First, find out what the L2 operator looks like in spherical coordinates. That'll simplify things quite a bit.

You used [itex]\hat{L}^2\psi = \hbar^2[l(l+1)]^2\psi[/itex], which is incorrect. If you have an eigenfunction [itex]\psi_{nlm}[/itex], then [itex]\hat{L^2}\psi_{nlm} = \hbar^2 l(l+1) \psi_{nlm}[/itex]. The factor l(l+1) isn't squared.

Where did those factors of 2 in front of each integral come from?
 
  • #5
Wikipedia says [itex]L^2 = -\hbar^2 \left(\frac{1}{\sin\theta}\frac{\partial}{\partial \theta}\left( \sin\theta \frac{\partial}{\partial \theta}\right) + \frac{1}{\sin^2\theta}\frac{\partial^2}{\partial \phi^2}\right)[/itex]. But that would be more complicated, because I don't see it go away by a derivation. [itex]\psi[/itex] is still dependent on the angles. Besides, there is an r² too much in front of [itex]\frac{\hbar^{4}l^{2}(l^{2}+2l+1)}{2\mu r^{2}}[/itex] in my solution.
An integral from -infinity to +infinity is two times the same integral from 0 to infinity.
 
  • #6
You seem to have some misconceptions related to spherical coordinates. You need to get those straightened out first, so you should go back and review the basics of spherical coordinates. Note that the vector [itex]\vec{r}[/itex] and the coordinate [itex]r[/itex] are not the same thing, so in the equations you find in your textbook, you need to understand which one is being referred to.

Say you have a generic wave function [itex]\psi[/itex]. In Cartesian coordinates, the quantity [itex]\langle \psi \vert \psi \rangle[/itex] would be given by[tex]\langle \psi \vert \psi \rangle = \int_{-\infty}^\infty \int_{-\infty}^\infty \int_{-\infty}^\infty \psi^*(x,y,z)\psi(x,y,z)\,dx\,dy\,dz[/tex]How would you express this quantity using spherical coordinates?
 
  • #7
I'd say that is in spherical coordinates:
[itex]\langle \psi \vert \psi \rangle = \int_{-\infty}^\infty \int_{-\infty}^\infty \int_{-\infty}^\infty r²\cdot sin² (\theta) \psi^*(r, \theta,\phi)\psi(r,\theta,\phi)\ ,dr\,d \theta\,d\phi[/itex].

I understand of course that [itex]\vec{r}=(r cos \theta cos \phi, r cos \theta sin ..., cos) [/itex] or something like that. Still don't see the connection.
 
  • #8
Close, but you need to understand what values the coordinates can take on in each system. In Cartesian coordinate, you have to let x, y, and z each run from -∞ to +∞ to cover all of space. What about in spherical coordinates? Remember you don't want to cover a point multiple times. There should be only one combination of r, θ, and φ for each point in space.
 
  • #9
[tex]\langle \psi \vert \psi \rangle = \int_{0}^\infty \int_{0}^\pi \int_{0}^{2\pi} r²\cdot sin² (\theta) \psi^*(r, \theta,\phi) \psi(r,\theta,\phi)\ ,dr\,d \theta\, d\phi [/tex]
 
  • #10
That's correct except sin θ shouldn't be squared.

So now back to the problem. You have [itex]\psi = C e^{-\alpha r}[/itex], so[tex]\langle \psi \vert \psi \rangle = \int_0^\infty \int_0^\pi \int_0^{2\pi} e^{-2\alpha r}r^2\sin \theta \,dr\,d\theta\,d\phi[/tex]What do you get when you evaluate this integral?
 
  • #11
[itex]\langle \psi \vert \psi \rangle = \int_0^\infty \int_0^\pi \int_0^{2\pi} e^{-2\alpha r}r^2\sin \theta \,dr\,d\theta\,d\phi=\frac{1}{4}\alpha^{3} \cdot 2\pi²[/itex]
And with [tex]L^2 = -\hbar^2 \left(\frac{1}{\sin\theta}\frac{\partial}{\partial \theta}\left( \sin\theta \frac{\partial}{\partial \theta}\right) + \frac{1}{\sin^2\theta}\frac{\partial^2}{\partial \phi^2}\right)[/tex] the whole L term goes away?
 
  • #12
Juqon said:
[itex]\langle \psi \vert \psi \rangle = \int_0^\infty \int_0^\pi \int_0^{2\pi} e^{-2\alpha r}r^2\sin \theta \,dr\,d\theta\,d\phi=\frac{1}{4}\alpha^{3} \cdot 2\pi²[/itex]
That's not correct. Try again.
And with [tex]L^2 = -\hbar^2 \left(\frac{1}{\sin\theta}\frac{\partial}{\partial \theta}\left( \sin\theta \frac{\partial}{\partial \theta}\right) + \frac{1}{\sin^2\theta}\frac{\partial^2}{\partial \phi^2}\right)[/tex] the whole L term goes away?
Yes. Can you explain why?
 
  • #13
[tex]\langle \psi \vert \psi \rangle = \int_0^\infty \int_0^\pi \int_0^{2\pi} e^{-2\alpha r}r^2\sin \theta \,dr\,d\theta\,d\phi=\frac{1}{4}\alpha^{3} \cdot 2\pi \cdot\left[-cos\right]^\pi _0 = \frac{1}{4}\alpha^{3} \cdot 2\pi \cdot\left[-(-1) - (-(1))\right]=\frac{1}{4}\alpha^{3} \cdot 4\pi =\alpha^{3}\pi[/tex]
It goes away because it is differentiated with respect to [itex]\theta[/itex] and [itex]\varphi[/itex], but these are not in the result any more, is a constant with respect to the angles and thus 0.
 
  • #14
Excellent! Now onto the numerator...
 
  • #15
Thanks, now here is my newest problem.
[EDIT: I think I found it.][tex]E_{0}=\frac{<\psi|\hat{H}|\psi>}{<\psi|\psi>}= \frac{\int_{-\infty}^{\infty}dr\cdot r^{2}\cdot\psi^{*}\left\{ -\frac{\hbar^{2}}{2\mu}\left[\alpha^{2}-\frac{2}{r}\alpha\right]\psi+\frac{1}{4\pi\epsilon_{0}} \frac{qQ}{r}\psi\right\} }{\int_{-\infty}^{\infty}dr\psi^{*}\psi}[/tex]

[tex]

=\frac{2\int_{0}^{\infty}dr\cdot r^{2}\cdot\exp\left\{ -2\alpha r\right\} \left\{ \frac{\hbar^{2}}{2\mu}\frac{2}{r}\alpha-\frac{\hbar^{2}}{2\mu}\alpha^{2}+\frac{1}{4 \pi \epsilon_{0}}\frac{qQ}{r}\right\} }{2\int_{0}^{\infty}dr\cdot r^{2}\cdot\exp\left\{ -2\alpha r\right\} }
[/tex]

[tex]=\frac{2\int_{0}^{\infty}dr\cdot\exp\left\{ -2\alpha r\right\} \left\{ \frac{\hbar^{2}}{2\mu}2r\alpha-r^{2}\frac{\hbar^{2}}{2\mu}\alpha^{2}+\frac{r}{4 \pi \epsilon_{0}}\frac{qQ}{1}\right\} }{2\int_{0}^{\infty}dr\cdot r^{2}\cdot\exp\left\{ -2\alpha r\right\} }[/tex][tex]
=\frac{2\int dr\cdot\exp\left\{ -2\alpha r\right\} \cdot r\left\{ \frac{\hbar^{2}}{2\mu}2\alpha+\frac{1}{4 \pi \epsilon_{0}}\frac{qQ}{1}\right\} +2\int dr\cdot\exp\left\{ -2\alpha r\right\} \cdot r^{2}\cdot\left(-\frac{\hbar^{2}}{2\mu}\alpha^{2}\right)}{2\int_{0}^{\infty}dr\cdot r^{2}\cdot\exp\left\{ -2\alpha r\right\} }

[/tex]

[tex]
=\frac{2\cdot\frac{1}{4}\alpha^{2}\left\{ \frac{\hbar^{2}}{2\mu}2\alpha+\frac{1}{4 \pi \epsilon_{0}}\frac{qQ}{1}\right\} +2\frac{1}{4}\alpha^{3}\cdot\left(-\frac{\hbar^{2}}{2\mu}\alpha^{2}\right)}{2\frac{1}{4}\alpha^{3}}[/tex]

[tex]=\frac{\frac{1}{2}\alpha^{2}\left\{ \frac{\hbar^{2}}{2\mu}2\alpha+\frac{1}{4 \pi \epsilon_{0}}\frac{qQ}{1}\right\} +\frac{1}{2}\alpha^{3}\cdot\left(-\frac{\hbar^{2}}{2\mu}\alpha^{2}\right)}{\frac{1}{2}\alpha^{3}}

[/tex]

[tex]

E_{0}=\frac{1}{4 \pi \epsilon_{0}}\frac{qQ}{\alpha}+\frac{\hbar^{2}}{ \mu}

[/tex]

[tex]

\frac{\partial E_{0}}{\partial\alpha}=-\frac{1}{4 \pi \epsilon_{0}}\frac{qQ}{\alpha^{2}}=0[/tex]
 
Last edited:
  • #16
You don't see a disconnect between what you wrote in post 13 and what you wrote in post 15?
 
  • #17
This is my newest version. I think I showed in #13 what the result of the integral ist, but now that differentiations make it 0 I deleted the whole L term.

[tex]
E_{0}= \frac{<\psi|\hat{H}|\psi>}{<\psi|\psi>}=\frac{\int_{-\infty}^{\infty}dr\cdot r^{2}\cdot\psi^{*}\left\{ -\frac{\hbar^{2}}{2\mu}\left[\alpha^{2}-\frac{2}{r}\alpha\right]\psi-\frac{e^{2}}{r}\psi\right\} }{\int_{-\infty}^{\infty}dr\psi^{*}\psi}
[/tex]

[tex]


=\frac{2\int_{0}^{\infty}dr\cdot r^{2}\cdot\exp\left\{ -2\alpha r\right\} \left\{ \frac{\hbar^{2}}{2\mu}\frac{2}{r}\alpha-\frac{\hbar^{2}}{2\mu}\alpha^{2}-\frac{e^{2}}{r}\right\} }{2\int_{0}^{\infty}dr\cdot r^{2}\cdot\exp\left\{ -2\alpha r\right\} }
[/tex]

[tex]


=\frac{2\int_{0}^{\infty}dr\cdot\exp\left\{ -2\alpha r\right\} \left\{ \frac{\hbar^{2}}{2\mu}2r\alpha-r^{2}\frac{\hbar^{2}}{2\mu}\alpha^{2}-e^{2}\right\} }{2\int_{0}^{\infty}dr\cdot r^{2}\cdot\exp\left\{ -2\alpha r\right\} }
[/tex]

[tex]


=\frac{2\int dr\cdot\exp\left\{ -2\alpha r\right\} \cdot r\left\{ \frac{\hbar^{2}}{2\mu}2\alpha-e^{2}\right\} +2\int dr\cdot\exp\left\{ -2\alpha r\right\} \cdot r^{2}\cdot\left(-\frac{\hbar^{2}}{2\mu}\alpha^{2}\right)}{2\int_{0}^{\infty}dr\cdot r^{2}\cdot\exp\left\{ -2\alpha r\right\} }

[/tex]

[tex]

=\frac{2\cdot\frac{1}{4}\alpha^{2}\left\{ \frac{\hbar^{2}}{2\mu}2\alpha-e^{2}\right\} +2\frac{1}{4}\alpha^{3}\cdot\left(-\frac{\hbar^{2}}{2\mu}\alpha^{2}\right)}{2\frac{1}{4}\alpha^{3}}

[/tex]

[tex]

=\frac{\frac{1}{2}\alpha^{2}\left\{ \frac{\hbar^{2}}{2\mu}2\alpha-e^{2}\right\} +\frac{1}{2}\alpha^{3}\cdot\left(-\frac{\hbar^{2}}{2\mu}\alpha^{2}\right)}{\frac{1}{2}\alpha^{3}}

[/tex]

[tex]

E_{0}=-\frac{e^{2}}{\alpha}+\frac{\hbar^{2}}{\mu}-\frac{\hbar^{2}}{2\mu}\alpha^{2}
[/tex]

[tex]


\frac{\partial E_{0}}{\partial\alpha}=\frac{e^{2}}{\alpha^{2}}-\frac{\hbar^{2}}{\mu}\alpha=0\Leftrightarrow\frac{\hbar^{2}}{\mu}\alpha=\frac{e^{2}}{\alpha^{2}}

[/tex]

[tex]

\Leftrightarrow\alpha^{3}=e^{2}\cdot\frac{\mu}{ \hbar^{2}}\Leftrightarrow\alpha=\sqrt[3]{e^{2}\cdot\frac{\mu}{\hbar^{2}}}
[/tex]

[tex]


E_{0}=- \frac{e^{2}}{\sqrt[3]{e^{2}\cdot\frac{\mu}{\hbar^{2}}}} + \frac{\hbar^{2}}{\mu}-\frac{\hbar^{2}}{2\mu} \sqrt[3]{e^{2}\cdot\frac{\mu}{\hbar^{2}}}^{2}
[/tex]
 
  • #18
Look at the denominators in the very first line. How is that consistent with what you have in post #13?
 
  • #19
Mh, I didn't change the other integrals.

[tex]

E_{0}= \frac{<\psi|\hat{H}|\psi>}{<\psi|\psi>}=\frac{\int_{-\infty}^{\infty}d\vec{r}\psi^{*}\left\{ -\frac{\hbar^{2}}{2\mu}\left[\alpha^{2}-\frac{2}{r}\alpha\right]\psi-\frac{e^{2}}{r}\psi\right\} }{\int_{-\infty}^{\infty}d\vec{r}\psi^{*}\psi}[/tex]

[tex]


=\frac{\int_{-\infty}^{\infty}d\vec{r}\exp\left\{ -2\alpha r\right\} \left\{ \frac{\hbar^{2}}{2\mu}\frac{2}{r}\alpha-\frac{\hbar^{2}}{2\mu}\alpha^{2}-\frac{e^{2}}{r}\right\} }{\int_{-\infty}^{\infty}d\vec{r}\exp\left\{ -2\alpha r\right\} }[/tex]

[tex]


=\frac{\int_{0}^{\infty}dr \int_{0}^{\pi}d\theta\int_{0}^{2\pi}d\phi\cdot\sin\theta\cdot\exp\left\{ -2\alpha r\right\} \cdot r\left\{ \frac{\hbar^{2}}{2\mu}2\alpha-e^{2}\right\} +\int_{0}^{\infty}dr\int_{0}^{\pi}d\theta\int_{0}^{2\pi}d\phi\cdot\sin\theta\cdot\exp\left\{ -2\alpha r\right\} \cdot r^{2}\cdot\left(-\frac{\hbar^{2}}{2\mu}\alpha^{2}\right)}{\int_{0}^{\infty}dr\int_{0}^{\pi}d\theta\int_{0}^{2\pi}d \phi\cdot r^{2}\sin\theta\cdot\exp\left\{ -2\alpha r\right\} }[/tex]

[tex]


=\frac{\pi\alpha^{2}\left\{ \frac{\hbar^{2}}{\mu}\alpha-e^{2}\right\} +\pi\alpha^{3}\cdot\left(-\frac{\hbar^{2}}{2\mu}\alpha^{2}\right)}{\pi\alpha^{3}}[/tex]

[tex]


E_{0}=-\frac{e^{2}}{\alpha}+\frac{\hbar^{2}}{\mu}\left(1-\frac{\alpha^{2}}{2}\right)[/tex]

[tex]


[/tex] [tex] \frac{ \partial E_{0}}{\partial\alpha}=\frac{e^{2}}{\alpha^{2}}-\frac{\hbar^{2}}{\mu}2\alpha=0 \Leftrightarrow\frac{\hbar^{2}}{\mu}2\alpha=\frac{e^{2}}{\alpha^{2}}[/tex]

[tex]


[/tex] [tex]\Leftrightarrow \alpha^{3}=e^{2}\cdot\frac{\mu}{2\hbar^{2}} \Leftrightarrow\alpha=\sqrt[3]{e^{2}\cdot\frac{\mu}{2 \hbar^{2}}}[/tex]

[tex]


E_{0}=-\frac{e^{2}}{\sqrt[3]{e^{2}\cdot\frac{\mu}{2\hbar^{2}}}}+\frac{\hbar^{2}}{\mu2}\left(2-\sqrt[3]{e^{2}\cdot\frac{\mu}{2\hbar^{2}}}^{2}\right) [/tex]
 
Last edited:
  • #20
The integrals should be
\begin{align*}
\int_0^\infty r e^{-2\alpha r}\,dr = \frac{1}{4\alpha^2} &\\
\int_0^\infty r^2 e^{-2\alpha r}\,dr = \frac{1}{4\alpha^3} &
\end{align*}
With those corrections, it should work out.
 
  • #21
You are right, the integrals were wrong, thank you. This is my calculation with the right ones:
[tex] E_{0}=\frac{\frac{\pi}{\alpha^{2}}\left\{ \frac{\hbar^{2}}{\mu}\alpha-e^{2}\right\} +\frac{\pi}{\alpha^{3}}\cdot\left(-\frac{\hbar^{2}}{2\mu}\alpha^{2}\right)}{\frac{\pi}{\alpha^{3}}} [/tex]

[tex] E_{0}=\frac{\hbar^{2}}{\mu}\alpha^{2}\left(1-\frac{1}{2}\right)-e^{2}\alpha [/tex]

[tex] E_{0}=\frac{\hbar^{2}}{2\mu}\alpha^{2}-e^{2}\alpha [/tex]

[tex] \frac{\partial E_{0}}{\partial\alpha}=\frac{\hbar^{2}}{\mu}\alpha-e^{2}=0\Leftrightarrow\frac{\hbar^{2}}{\mu}\alpha=e^{2} [/tex]
[tex] \Leftrightarrow\alpha=e^{2}\cdot\frac{\mu}{\hbar^{2}} [/tex]

[tex] E_{0}=\frac{\hbar^{2}}{2\mu}\left(e^{2}\cdot\frac{\mu}{\hbar^{2}}\right)^{2}-e^{2}\left(e^{2}\cdot\frac{\mu}{\hbar^{2}}\right) [/tex]

[tex] E_{0}=\frac{\hbar^{2}}{2\mu}e^{4}\cdot\frac{\mu^{2}}{\hbar^{4}}-e^{4}\cdot\frac{\mu}{\hbar^{2}} [/tex]
[tex] =\left( \frac{\mu }{2 \hbar^{2} }- \frac{\mu}{\hbar^{2} }\right) \cdot e^{4} [/tex] [tex] =\frac{\mu}{\hbar^{2}} \left( \frac{1}{2}-1\right) \cdot e^{4}=-\frac{\mu e^{4}} {2 \hbar^{2}}[/tex]
 

1. What is zero-point energy?

Zero-point energy is the lowest possible energy that a quantum mechanical physical system can possess, even at absolute zero temperature. It is the energy that remains in a system when all other forms of energy, such as kinetic or thermal energy, have been removed.

2. How is zero-point energy calculated using the Schrödinger equation?

The Schrödinger equation is a fundamental equation in quantum mechanics that describes the behavior of a quantum system over time. The equation can be used to calculate the zero-point energy of a system by solving for the lowest possible energy state of the system.

3. What is the Ritz variational principle?

The Ritz variational principle is a mathematical approach for finding approximate solutions to the Schrödinger equation. It involves using a trial wavefunction, which is an educated guess for the true wavefunction of the system, and minimizing the energy of the system with respect to this trial wavefunction.

4. Why is the Ritz variational principle commonly used to calculate zero-point energy?

The Ritz variational principle is commonly used to calculate zero-point energy because it allows for a more efficient and accurate approximation of the true wavefunction compared to other methods. This is because it takes into account the variation of the wavefunction, rather than just the energy, which leads to more accurate results.

5. What are some real-world applications of zero-point energy?

Although zero-point energy cannot be harnessed directly, it plays a crucial role in various real-world applications. These include the stability of atoms and molecules, the behavior of materials at low temperatures, and the study of quantum phenomena such as superconductivity and superfluidity.

Similar threads

  • Advanced Physics Homework Help
Replies
3
Views
363
  • Advanced Physics Homework Help
Replies
10
Views
1K
  • Advanced Physics Homework Help
Replies
1
Views
864
  • Advanced Physics Homework Help
Replies
19
Views
801
  • Advanced Physics Homework Help
Replies
30
Views
1K
  • Advanced Physics Homework Help
Replies
5
Views
1K
Replies
1
Views
349
  • Advanced Physics Homework Help
Replies
1
Views
482
  • Advanced Physics Homework Help
Replies
0
Views
651
  • Advanced Physics Homework Help
Replies
24
Views
778
Back
Top